Почему точки Лагранжа L1L1L_1, L2L2L_2 и L3L3L_3 нестабильны?

Почему точки Лагранжа л 1 , л 2 & л 3 нестабильный? Я делаю презентацию по физике перед своим классом, и я просто подтверждаю, что это правильно, и если это не так, может ли кто-нибудь дать объяснение.

Ф "=" м в 2 / р и в "=" 2 π / Т

Поэтому:

Ф "=" 4 π 2 м р / Т 2

Ф 1 / Т 2 пока р постоянно.

Так что если р перестанет находиться в той же точке, что и точка Лагранжа, то изменится период обращения и система сместится.

Ответы (3)

I) ОП спрашивает (v5):

Почему точки Лагранжа л 1 , л 2 & л 3 нестабильный?

На самом деле, это отличный и нетривиальный вопрос. Не следует делать выводы, основываясь только на эффективном потенциале В (состоящий из силы тяжести и центробежного потенциала ) в одиночку, потому что сила Кориолиса может быть важна.

Например, глобальные максимальные баллы л 4 & л 5 из В (которые по наивности были бы последними местами, в которых можно было бы заподозрить устойчивую точку равновесия), иногда бывают устойчивыми, ср. например, этот пост Phys.SE!

II) Проанализируем здесь устойчивость произвольной точки с малой начальной скоростью в 0 в орбитальной плоскости 1 р 2 С при наличии силы Кориолиса. Давайте выберем координаты г е С так, чтобы начальное положение было в начале координат г "=" 0 .

Начнем с изучения критических/стационарных точек , т.е. точек Лагранжа. Воспользуемся следующей теоремой, упомянутой в [3]. 1:

Теорема. Учитывая гессен ЧАС для эффективного потенциала В : р 2 р в точке Лагранжа. Позволять Ом быть угловой скоростью. Следующие 3 условия необходимы и достаточны для того, чтобы точка Лагранжа была стабильной:

  1. С   "="   дет ( ЧАС )     0.

  2. Б   "="   т р ( ЧАС ) + 4 Ом 2     0.

  3. Д   "="   Б 2 4 С     0.

NB: Теорема игнорирует члены более высокого порядка в В , что может стать важным, если дет ( ЧАС ) "=" 0 .

Доказательство теоремы приведено в моем ответе Phys.SE здесь , где также обсуждается случай локального максимума.

  1. Случай локального минимума 2 дет ( ЧАС ) 0 т р ( ЧАС ) 0 : Он стабилен, потому что все 3 условия выполнены. Чтобы увидеть третье условие, заметьте, что

    (1) Д   "="   Б 2 4 С   "="   ( т р ( ЧАС ) + 4 Ом 2 ) 2 4 дет ( ЧАС )   "="   т р ( ЧАС ) 4 дет ( ЧАС ) + 8 т р ( ЧАС ) Ом 2 + 16 Ом 4   "="   ( ЧАС 11 ЧАС 22 ) 2 + 4 ЧАС 12 2 + 8 т р ( ЧАС ) 0 Ом 2 + 16 Ом 4   >   0.

  2. Случай сильной седловой точки дет ( ЧАС ) < 0 : Это случай точек Лагранжа л 1 , л 2 & л 3 , о чем спрашивает ОП. Он нестабилен, так как нарушает первое условие.

III) Чтобы получить некоторое представление о приведенных выше выводах, давайте также проанализируем, что происходит вдали от точек Лагранжа.

  1. Случай некритической точки: удельная действующая сила ф "=" В | г "=" 0 0 не равно нулю. Тогда в области, где удельную эффективную силу можно считать постоянной, ЭОМ с удельной силой Кориолиса 2 в × Ом читает 3

    (2) г ¨     2 я Ом г ˙ Кориолис + ф   г г т ( опыт { 2 я Ом т } г ˙ )     опыт { 2 я Ом т } ф .
    Отсюда скорость
    (3) г ˙     ф 2 я Ом + ( в 0 ф 2 я Ом ) опыт { 2 я Ом т } ,
    и позиция
    (4) г     ф т 2 я Ом дрейф + 1 2 я Ом ( ф 2 я Ом в 0 ) комплексная амплитуда ( опыт { 2 я Ом т } круг 1 )
    (5)     в 0 т + ( ф 2 я Ом в 0 ) т 2 2 ( 1 2 я Ом т 3 ) + О ( т 4 ) .
    Пробная частица совершает круговое движение с дрейфом вдоль эквипотенциальных линий в плоскости орбиты. (К р я г час т если вы ищете г о ж н час я л л .) В любом случае пробная частица неустойчива в некритической точке.

    Рис. 1: Возможная подковообразная орбита вдоль эквипотенциальных линий.

Приведенное выше описание позволяет понять, почему локальные минимумы и (иногда) локальные максимумы стабильны, а седловые точки всегда нестабильны. Если пробная частица стремится дрейфовать по эквипотенциальным линиям, то вблизи локальных минимумов и максимумов она попадает в небольшие концентрические петли, а эквипотенциальные линии всегда направлены в сторону от седловых точек.

Рис. 2: Пробная частица стремится дрейфовать вдоль эквипотенциальных линий из-за силы Кориолиса.

Использованная литература:

  1. Дж. Бинни и С. Тремейн, Galactic Dynamics, 2-е издание (2008 г.); п. 181-182.

--

1 Также могло иметь место колебательное движение перпендикулярно плоскости орбиты, которое мы игнорируем.

2 Вышеприведенный анализ справедлив для любого эффективного потенциала В . В ограниченной задаче трех тел, о которой спрашивает ОП, эффективный потенциал В не имеет локального минимума.

3 Здесь мы выбираем ориентацию угловой скорости Ом быть прямым/против часовой стрелки/положительным направлением вращения.

Я считаю, что самый интуитивный способ думать о точках Лагранжа — это рассматривать гравитационную потенциальную энергию и поверхность, представляющую этот потенциал.

Точки Лагранжа существуют в пространстве вокруг двух массивных объектов, скажем, Солнца и Земли. Вы можете думать об обоих из них как о создании гравитационного потенциала, хорошо имеющего форму конуса или воронки, направленной вниз, но со сторонами, образующими изогнутый профиль. Гравитационный потенциал как функция расстояния р дан кем-то U "=" г м / р . Вот красивая картинка, взятая из Википедии 2 . Источник изображения: AllenMcC. - Собственная работа, CC BY-SA 3.0, Graviational Potential WellГравитационная потенциальная скважина

Теперь, поскольку есть два массивных тела, рядом с каждым находится два таких колодца, хотя тот, который находится в центре более массивного тела, будет больше. Вы можете себе представить, что между двумя колодцами будет седловая точка, где поверхность будет плоской. Это будет представлять собой точку, в которой силы гравитации, притягивающие к каждому телу, находятся в равновесии (я не смог найти хорошее изображение двух колодцев).

Дело в том, что меньшее тело (скажем, Земля) вращается вокруг большего тела (Солнца) или, точнее, они оба вращаются вокруг своего общего барицентра. Точки Лагранжа неподвижны, если смотреть с Земли, поэтому нам нужно рассматривать вещи из системы отсчета, которая вращается вместе с Землей. В такой вращающейся системе отсчета мы должны ввести фиктивную центробежную силу вдали от центра вращения. Добавление этой силы изменяет потенциальную поверхность с гравитационными ямами. Поскольку сила действует вдали от центра, объект рядом с центром будет иметь более высокий потенциал, чем объект, находящийся дальше, поэтому потенциал этой силы будет выглядеть как перевернутая чаша. Когда вы объединяете эффекты гравитации обоих тел и центробежной силы, вы получаете несколько сложную эффективную потенциальную поверхность. Здесь'Эквипотенциал лаграновых точек

Источник изображения: Пользователь: cmglee - собственная работа, CC BY-SA 3.0, Лаграновы точки эквипотенциала

Итак, пять точек Лагранжа — это места, где наклон этой потенциальной поверхности равен нулю. Вы можете представить, что сидите там и не ускользаете от них, если будете осторожны. л 1 , л 2 , л 3 , имеют седловидную форму, тогда как л 4 и л 5 точки приходятся на локальные максимумы. Они похожи на вершины холмов, но довольно плоские вершины холмов.

Тем не менее, л 4 и л 5 точки не являются локальными минимумами, так как же они стабильны? Что ж, во вращающейся системе отсчета центробежная сила — не единственная фиктивная сила, которую необходимо ввести. Как только что-то начинает двигаться, на него также действует сила Кориолиса, перпендикулярная его движению. Если спутник удаляется от л 4 или л 5 указывает, что именно эта сила Кориолиса подтолкнет его обратно, если не точно обратно в точку Лагранжа, то на орбиту вокруг нее.

Я только понял, что неправильно объяснил - почему L4 и L5 стабильны. Я отредактирую.
В такой статье, как «Стабильность точек Лагранжа: космический телескоп Джеймса Уэбба», дается анализ стабильности всех пяти точек Лагранжа, но я предполагаю, что это выходит за рамки того уровня, который искал ОП. Есть ли более простой способ ответить на исходный вопрос, помимо того, что просто сказать, что эффективный потенциал не является локальным минимумом в рассматриваемых точках, а объяснение силы Кориолиса, которое мы используем для L4 и L5, не работает для L1, L2 и L3?

В вашем анализе отсутствуют два ключевых элемента: сила тяжести и то, как эта сила (и центробежная сила) зависит от r. Вам нужны те. Войдите во вращающуюся систему отсчета вокруг центра масс системы и добавьте гравитацию обоих объектов, а также центробежную силу (она существует, потому что мы находимся во вращающейся системе отсчета). Вы должны обнаружить, что сумма всех этих сил равна нулю в точках L1 и L3. Затем обратите внимание, что сила направлена ​​в сторону от точек L1 и L3, если вы немного измените ra. Если у вас нет математических навыков, чтобы увидеть это, вы можете просто сказать: объект прямо на L1 или L3 может обращаться с тем же орбитальным периодом, что и источники гравитации на орбите, но любое отклонение от это портит то. Это то, что вы сказали, но вы никогда не уточнили задействованные силы, поэтому недостаточно показать, что он нестабилен, потому что неустойчивость действительно зависит от этих сил. Насколько вы хотите это показать или просто сказать, зависит от глубины вашей презентации.